Menu Close

Find-four-values-of-n-satisfying-1-n-2000-and-2-n-n-2-mod-1024-




Question Number 111503 by Aina Samuel Temidayo last updated on 04/Sep/20
Find four values of n satisfying  1≤n≤2000 and 2^n =n^2 (mod 1024)
$$\mathrm{Find}\:\mathrm{four}\:\mathrm{values}\:\mathrm{of}\:\mathrm{n}\:\mathrm{satisfying} \\ $$$$\mathrm{1}\leqslant\mathrm{n}\leqslant\mathrm{2000}\:\mathrm{and}\:\mathrm{2}^{\mathrm{n}} =\mathrm{n}^{\mathrm{2}} \left(\mathrm{mod}\:\mathrm{1024}\right) \\ $$
Answered by 1549442205PVT last updated on 04/Sep/20
2^n =n^2 (mod 1024)⇔2^n −n^2 =k.2^(10)   ⇔n^2 =2^n −k.2^(10) =2^(10) (2^(n−10) −k)(k∈N^∗ )  ⇔((n/(32)))^2 =2^(n−10) −k.⇒n=32m(m∈N^∗ )  1<n≤2000⇒32m<2000⇒1≤m≤62  ⇒m^2 =2^(32m−10) −k(1)  m=1⇒n=32,k=2^(22) −1  m=2⇒n=64,k=2^(54) −4  m=3⇒n=96,k=2^(86) −9  m=4⇒n=128,k=2^(118) −16  Thus ,we found four values of n  satisfying 2^n =n^2 (mod 1024)are  n∈{32,64,96,128}
$$\mathrm{2}^{\mathrm{n}} =\mathrm{n}^{\mathrm{2}} \left(\mathrm{mod}\:\mathrm{1024}\right)\Leftrightarrow\mathrm{2}^{\mathrm{n}} −\mathrm{n}^{\mathrm{2}} =\mathrm{k}.\mathrm{2}^{\mathrm{10}} \\ $$$$\Leftrightarrow\mathrm{n}^{\mathrm{2}} =\mathrm{2}^{\mathrm{n}} −\mathrm{k}.\mathrm{2}^{\mathrm{10}} =\mathrm{2}^{\mathrm{10}} \left(\mathrm{2}^{\mathrm{n}−\mathrm{10}} −\mathrm{k}\right)\left(\mathrm{k}\in\mathbb{N}^{\ast} \right) \\ $$$$\Leftrightarrow\left(\frac{\mathrm{n}}{\mathrm{32}}\right)^{\mathrm{2}} =\mathrm{2}^{\mathrm{n}−\mathrm{10}} −\mathrm{k}.\Rightarrow\mathrm{n}=\mathrm{32m}\left(\mathrm{m}\in\mathbb{N}^{\ast} \right) \\ $$$$\mathrm{1}<\mathrm{n}\leqslant\mathrm{2000}\Rightarrow\mathrm{32m}<\mathrm{2000}\Rightarrow\mathrm{1}\leqslant\mathrm{m}\leqslant\mathrm{62} \\ $$$$\Rightarrow\mathrm{m}^{\mathrm{2}} =\mathrm{2}^{\mathrm{32m}−\mathrm{10}} −\mathrm{k}\left(\mathrm{1}\right) \\ $$$$\mathrm{m}=\mathrm{1}\Rightarrow\mathrm{n}=\mathrm{32},\mathrm{k}=\mathrm{2}^{\mathrm{22}} −\mathrm{1} \\ $$$$\mathrm{m}=\mathrm{2}\Rightarrow\mathrm{n}=\mathrm{64},\mathrm{k}=\mathrm{2}^{\mathrm{54}} −\mathrm{4} \\ $$$$\mathrm{m}=\mathrm{3}\Rightarrow\mathrm{n}=\mathrm{96},\mathrm{k}=\mathrm{2}^{\mathrm{86}} −\mathrm{9} \\ $$$$\mathrm{m}=\mathrm{4}\Rightarrow\mathrm{n}=\mathrm{128},\mathrm{k}=\mathrm{2}^{\mathrm{118}} −\mathrm{16} \\ $$$$\mathrm{Thus}\:,\mathrm{we}\:\mathrm{found}\:\mathrm{four}\:\mathrm{values}\:\mathrm{of}\:\mathrm{n} \\ $$$$\mathrm{satisfying}\:\mathrm{2}^{\mathrm{n}} =\mathrm{n}^{\mathrm{2}} \left(\mathrm{mod}\:\mathrm{1024}\right)\mathrm{are} \\ $$$$\mathrm{n}\in\left\{\mathrm{32},\mathrm{64},\mathrm{96},\mathrm{128}\right\} \\ $$
Commented by Aina Samuel Temidayo last updated on 04/Sep/20
Thanks.
$$\mathrm{Thanks}. \\ $$

Leave a Reply

Your email address will not be published. Required fields are marked *